Answered

Obtenez les meilleures solutions à toutes vos questions sur Laurentvidal.fr, la plateforme de Q&R de confiance. Connectez-vous avec des professionnels prêts à fournir des réponses précises à vos questions sur notre plateforme complète de questions-réponses. Découvrez la facilité d'obtenir des réponses rapides et précises à vos questions grâce à l'aide de professionnels sur notre plateforme.

Bonsoir pouvez-vous m'aider svp?

Bonsoir Pouvezvous Maider Svp class=

Sagot :

Réponse :

1. [tex]n \leq \frac{ln(0.33)}{ln(10^{-5})}[/tex]

2. [tex]n \leq \frac{ln(0.5)}{ln(0.003)}[/tex]

3. [tex]n \leq \frac{ln(0.77)}{ln(0.001)}[/tex]

4. [tex]n \geq \frac{ln(\frac{10}{3} )}{ln(1.045)}[/tex]

5. [tex]n < \frac{ln(0.1)}{ln(0.8)}[/tex]

Explications étape par étape :

Pour résoudre les inéquations il faut réussir à isoler le terme xⁿ pour pouvoir utiliser la fonction logarithme népérien.

Cela va t'aider car ㏑(xⁿ) = n×㏑(x). De plus la fonction ㏑ est croissante sur N donc elle ne change pas le sens de l'inégalité.

1. [tex]0,33^n \leq 10^{-5}[/tex] ⇔  [tex]ln(0,33^n) \leq ln(10^{-5})[/tex] ⇔  [tex]n\times ln(0.33) \leq ln(10^{-5})[/tex]

donc   [tex]n \leq \frac{ln(0.33)}{ln(10^{-5})}[/tex]

2. [tex]0,5^n \leq 0.003[/tex]  ⇔  [tex]ln(0,5^n) \leq ln(0.003)[/tex]  ⇔  [tex]n\times ln(0.5) \leq ln(0.003)[/tex]

donc [tex]n \leq \frac{ln(0.5)}{ln(0.003)}[/tex]

3. [tex]1-0.77^n \geq 0.999[/tex] ⇔ [tex]-0.77^n \geq 0.999-1[/tex]  ⇔  [tex]-0.77^n \geq -0.001[/tex]

On multiplie l'inégalité par -1 pour pouvoir utiliser la fonction ln parce que rappel, l'ensemble de définition de ln c'est ] 0 ; + ∞ [.

D'où,  [tex]0.77^n \leq 0.001[/tex]  ⇔  [tex]ln(0.77^n) \leq ln(0.001)[/tex] ⇔ [tex]n\times ln(0.77) \leq ln(0.001)[/tex]

Donc [tex]n \leq \frac{ln(0.77)}{ln(0.001)}[/tex]

4. [tex]3 \times 1.045^n \geq 10[/tex] ⇔ [tex]1.045^n \geq \frac{10}{3}[/tex] ⇔ [tex]ln(1.045^n)\geq ln(\frac{10}{3})[/tex] ⇔[tex]n\times ln(1.045) \geq ln(\frac{10}{3} )[/tex]

donc [tex]n \geq \frac{ln(\frac{10}{3} )}{ln(1.045)}[/tex]

5. [tex]1 - 0,8^n > 0.9[/tex] ⇔ [tex]-0.8^n > 0.9 - 1[/tex] ⇔ [tex]-0.8^n > -0.1[/tex] ⇔ [tex]0.8^n < 0.1[/tex]

D'où, [tex]ln(0.8^n) < ln(0.1)[/tex] ⇔  [tex]n\times ln(0.8) < ln(0.1)[/tex]

Donc [tex]n < \frac{ln(0.1)}{ln(0.8)}[/tex]

Merci d'utiliser notre service. Nous sommes toujours là pour fournir des réponses précises et à jour à toutes vos questions. Nous espérons que cela vous a été utile. Revenez quand vous voulez pour obtenir des réponses plus précises et des informations à jour. Revenez sur Laurentvidal.fr pour obtenir plus de connaissances et de réponses de nos experts.